You are on page 1of 27

ART 428

1) AYALA LAND VS RAY BURTON 29 SCRA 48

FACTS:

On March 20, 1984, KARAMFIL bought from AYALA a piece of land. The transaction was documented in a Deed
of Sale of even date, with certain special conditions and restrictions on the use or occupancy of the land. The said
special conditions and restrictions were attached as an annex to the deed of sale and incorporated in the
"Memorandum of Encumbrances" at the reverse side of the title of the lot. On February 18, 1988, KARAMFIL
sold the lot to PALMCREST. This deed was submitted to AYALA for approval. AYALA gave its written
conformity to the sale but reflecting in its approval the same special conditions/restrictions as in the previous
sale.

PALMCREST in turn sold the lot to Ray Burton Development Corporation (RBDC), now respondent, on April 11,
1988. Sometime in June of 1989, RBDC submitted to AYALA for approval a set of architectural plans for the
construction of a 5-storey office building on the subject lot. Since the building was well within the 42-meter height
restriction, AYALA approved the architectural plans.

The September 21, 1990 issue of the Business World magazine featured the "Trafalgar Plaza" as a modern 27-
storey structure which will soon rise in Salcedo Village, Makati City. Stunned by this information, AYALA,
through counsel, then sent a letter to RBDC demanding the latter to cease the construction of the building which
dimensions do not conform to the previous plans it earlier approved.

After trial on the merits, the trial court rendered a Decision on April 28, 1994 in favor of RBDC. Dissatisfied,
AYALA appealed to the Court of Appeals which affirmed the judgment of the trial court

ISSUE: Whether or not the trial court and the appellate court erred in rendering judgement in favor of RBDC.

HELD: The erroneous annotation of the 23-meter height restriction in RBDC's title was explained by Jose
Cuaresma, AYALA's Assistant Manager for Marketing and Sales. Jose Cuaresma further clarified that the
correct height restriction imposed by AYALA on RBDC was 42 meters. This height ceiling, he said, is based on
the deed of restrictions attached as annex to the deed of sale, and the same has been uniformly imposed on the
transferees beginning from the original deed of sale between AYALA and KARAMFIL.

This clarificatory statement of Jose Cuaresma should have cautioned the Court of Appeals from making the
unfounded and sweeping conclusion that RBDC can do anything it wants on the subject property "as if no
restrictions and conditions were imposed thereon," What is clear from the record, however, is that RBDC was the
party guilty of misrepresentation and/or concealment when it resorted to the fraudulent scheme of submitting
two (2) sets of building plans, one (1) set conformed to the Deed Restrictions, which was submitted to and
approved by AYALA, while another set violated the said restrictions, and which it presented to the Makati City
Building Official in order to secure from the latter the necessary building permit.

It is noteworthy that after the submission of the second set of building plans to the Building Official, RBDC
continued to make representations to AYALA that it would build the five-storey building in accordance with the
first set of plans approved by AYALA, AYALA relied on RBDC's false representations and released the said title.
Hence, RBDC was in bad faith.

In the instant case, the stipulations in the Deed Restrictions and Special Conditions are plain and unambiguous
which leave no room for interpretation

2) FAJARDO VS. FREEDOM TO BUILD

ELISEO FAJARDO JR., VS FREEDOM TO BUILD INC.

G. R. No. 134692 August 1, 2000

FACTS:

|
Freedom to Build Inc., an owner-developer and seller of low-cost housing sold to petitioner-spouses a house and
lot in the De La Costa Homes, in Barangka, Marikina, Metro Manila. The Contract to sell executed between the
parties, contained a Restrictive Covenant providing certain prohibitions, to wit:

“Easements. For the good of the entire community, the homeowner must observe a two-meter easement in front.
No structure of any kind (store, garage, bodega, etc.) may be built on the front easement.

“Upward expansion. A second storey is not prohibited. But the second storey expansion must be placed above the
back portion of the house and should not extend forward beyond the apex of the original building.

“Front expansion: 2nd Storey: No unit may be extended in the front beyond the line as designed and implemented
by the developer in the 60 sq. m. unit. In other words, the 2nd floor expansion, in front, is 6 meters back from the
front property line and 4 meters back from the front wall of the house, just as provided in the 60 sq. m. units.”

The above restrictions were also contained in Transfer Certificate of Title No. N-115384 covering the lot issued in
the name of petitioner-spouses.

The controversy arose when the petitioners despite repeated demand from the respondent, extended the roof of
their house to the property line and expanded the second floor of their house to a point directly above the original
front wall. Respondent filed before the RTC an action to demolish the unauthorized structures.

The RTC rendered a judgment against the petitioner ordering them to immediately demolish and remove the
extension of their expanded housing unit that exceeds the limitations imposed by the Restrictive Covenant,
otherwise the Branch Sheriff of this Court will execute the this decision at the expense of the defendants.

On appeal, the CA affirmed the decision of the RTC. Hence, this petition for review.

ISSUE: Whether or not the for the lack of a specific provision, prescribing the penalty of the demolition in the
“Restrictive Covenant” in the event of the breach thereof, the prayer of the respondent to demolish the structure
should fail.

RULING:

The Court held that the argument of the petitioner-spouses has no merit; Article 1168 of the New Civil Code
states that: “When the obligation consists in not doing and the obligor does what has been forbidden him, it shall
be undone at his expense.”

This Court is not unaware of its ruling in Ayala Corporation vs. Ray Burton Development Corporation, which has
merely adjudged the payment of damages in lieu of demolition.

In the aforementioned case, however, the elaborate mathematical formula for the determination of compensatory
damages which takes into account the current construction cost index during the immediately preceding 5 years
based on the weighted average of wholesale price and wage indices of the National Census and Statistics Office
and the Bureau of Labor Statistics is explicitly provided for in the Deed of Restrictions entered into by the
parties. This unique and peculiar circumstance, among other strong justifications therein mentioned, is not
extant in the case at bar.

In sum, the Court holds that since the extension constructed exceeds the floor area limits of the Restrictive
Covenant, petitioner spouses can be required to demolish the structure to the extent that it exceeds the
prescribed floor area limits.

Wherefore, the assailed decision of the Court of Appeals is AFFIRMED. No costs.

3) TAYAG VS LACSON

FACTS:

|
In March 1996 a group of farmer-tenants on three parcels of land owned by the Lacsons assigned to petitioner
Tayag their rights as tenants/tillers for p50/sqm.

The said amount would be payable “when the legal impediments to the sale of the property to the petitioner no
longer existed.” Tayag would have exclusive rights to purchase the property if and when the Lacsons agreed to
sell the property.

Tayag gave varied sums of money to the farmers as partial payments, and the farmers issued receipts.

Sometime later Tayag discovered that the farmers changed their minds and would be selling their rights to the
Lacsons instead, prompting Tayag to pray for Injunction against the farmers and Lacson.

In their defense, the Lacsons claimed that they did not induce the farmers to violate their contracts with Tayag,
and that since the farmers were merely tenants, they had no right to enter into any transactions involving Lacson
properties without the owners’ consent.

ISSUE: WON there was a valid option contract between Tayag and the farmers by virtue of the deeds of
assignment. NO

HELD:

Option contract defined

An option is a contract by which the owner of the property agrees with another person that he shall have the
right to buy his property at a fixed price within a certain time or under, or in compliance with certain terms and
conditions, or which gives to the owner of the property the right to sell or demand a sale. It imposes no binding
obligation on the person holding the option, aside from the consideration for the offer. Until accepted, it is not,
properly speaking, treated as a contract.

The second party gets in praesenti, not lands, not an agreement that he shall have the lands, but the right to call
for and receive lands if he elects.

Until accepted, it is not, properly speaking, treated as a contract. An option contract is a separate and distinct
contract from which the parties may enter into upon the conjunction of the option.

Farmers had no right to grant Tayag the option/right to buy the property as they were merely tenants

In this case, the defendants-tenants-subtenants, under the deeds of assignment, granted to the petitioner not
only an option but the exclusive right to buy the landholding. But the grantors were merely the defendants-
tenants, and not the respondents, the registered owners of the property. Not being the registered owners of the
property, the defendants-tenants could not legally grant to the petitioner the option, much less the "exclusive
right" to buy the property. Nemo dat quod non habet, literally meaning "no one gives what he doesn't have"
applies in this case.

Deeds of Assignment not valid; conditions stipulated did not arise

The full payment of 50/sqm under Tayag and the farmers’ ‘’option contracts’ were on the following conditions:

- that the Lacsons would agree to sell their property

- that the deeds of assignment were subject to the approval of DAR

- that there was a prohibitive period within which the farmers were able to sell their interest in the land

There is no showing in Tayag’s complaint that the farmers had agreed to sell their property, and that the legal
impediments to the agreement no longer existed. They had yet to submit the Deeds of Assignment to the
Department of Agrarian Reform which, in turn, had to act on and approve or disapprove the same. Unless the
DAR approves the deeds, Tayag has no right to enforce the same by asking the trial court to fix a period within
which to pay.

4) ROSS RICA SALES CENTER VS. SPS. ONG

|
ROSS RICA SALES CENTER vs. SPS. ONG

G.R. No. 132197. 08/16/2005, TINGA , J .

FACTS: The spouses Ong are the original owners of 3 parcels of land which they occupy. They sold it to Mandaue
Prime Estate Realty, which then sold it to Ross Rica Sales Center, Inc. The spouses Ong filed an action to annul
the sale and transfer of property to Mandaue Prime Estate Realty and at present, the case is still pending. In the
meantime, an ejectment case was filed against spouses Ong in the MTC, which ruled against the latter. On
appeal to the RTC, the judgment was affirmed by a decision dated March 1, 1997. The spouses Ong received a
copy of the decision on April 28, 1997.

The spouses Ong first filed a Notice of Appeal with the RTC (May 8, 1997) but on the very next day filed a Motion
for Reconsideration, which was denied on June 23, 1997. The spouses Ong received a copy of the order on July 9,
1997. On July 24, 1997 respondents filed with the CA a motion for an additional 10 days to file their Petition for
Review, which they would eventually file on July 30, 1997.

The CA gave their petition for review due course and reversed the decision of the RTC on the finding that the
action filed was not one for unlawful detainer based on two grounds: that the allegations fail to show that
petitioners were deprived of possession by force, intimidation, threat, strategy or stealth; and that there is no
contract, express or implied, between the parties that would qualify the case as one of unlawful detainer.

ISSUES/HELD

Whether the complaint satisfies the jurisdictional requirements for a case of unlawful detainer
properly cognizable by the MTC

- YES. Well-settled is the rule that what determines the nature of an action as well as which court has
jurisdiction over it are the allegations of the complaint and the character of the relief sought.

In Javelosa vs. Court of the Appeals, it was held that the allegation in the complaint that there was unlawful
withholding of possession is sufficient to make out a case for unlawful detainer.

It is equally settled that in an action for unlawful detainer, an allegation that the defendant is unlawfully
withholding possession from the plaintiff is deemed sufficient, without necessarily employing the terminology of
the law.

Hence, the phrase “unlawful withholding” has been held to imply possession on the part of defendant, which was
legal in the beginning, having no other source than a contract, express or implied, and which later expired as a
right and is being withheld by defendant.

In Rosanna B. Barba vs. Court of Appeals, the Supreme Court held that a simple allegation that the defendant is
unlawfully withholding possession from plaintiff is sufficient. Based on this premise, the allegation in the
Complaint that:

title and/or reconveyance, the rights asserted and the relief prayed for are not the same.

“. . . . despite demand to vacate, the defendants have refused and still refuse to vacate said lots, thus, unlawfully
withholding possession of said lots from plaintiffs and depriving plaintiffs of the use of their lots;” is already
sufficient to constitute an unlawful detainer case.

The long settled rule is that the issue of ownership cannot be the subject of a collateral attack. In Apostol vs.
Court of Appeals, this Court had the occasion to clarify this:

Likewise, the case of Co Tiamco vs. Diaz provides for a liberal approach in considering the sufficiency of a
complaint for unlawful detainer, thus:

“. . . The principle underlying the brevity and simplicity of pleadings in forcible entry and unlawful detainer cases
rests upon considerations of public policy. Cases of forcible entry and detainer are summary in nature, for they
involve perturbation of social order which must be restored as promptly as possible and, accordingly,
technicalities or details of procedure should be carefully avoided.”

|
Whether the case should be considered as one for accionreivindicatoria, and thus the jurisdiction
would lie with the RTC

- NO. The issue involved in accionreivindicatoria is the recovery of ownership of real property. This differs from
accionpubliciana where the issue is the better right of possession or possession de jure, and accioninterdictal
where the issue is material possession or possession de facto. In an action for unlawful detainer, the question of
possession is primordial, while the issue of ownership is generally unessential.

Petitioners, in all their pleadings, only sought to recover physical possession of the subject property. The mere
fact that they claim ownership over the parcels of land as well did not deprive the MTC of jurisdiction to try the
ejectment case.

Even if respondents claim ownership as a defense to the complaint for ejectment, the conclusion would be the
same, for mere assertion of ownership by the defendant in an ejectment case will not oust the municipal court of
its summary jurisdiction.

This Court in Ganadin vs. Ramos stated that if what is prayed for is ejectment or recovery of possession, it does
not matter if ownership is claimed by either party.

Therefore, the pending actions for Declaration of Nullity of Deed of Sale and Transfer Certificates of Title and
quieting of title in Civil Case No. MAN-2356 will not abate the ejectment case.

In Drilon vs. Gaurana, this Court ruled that the filing of an action for reconveyance of title over the same
property or for annulment of the deed of sale over the land does not divest the MTC of its jurisdiction to try the
forcible entry or unlawful detainer case before it, the rationale being that, while there may be identity of parties
and subject matter in the forcible entry case and the suit for annulment of

“. . . Under Section 48 of Presidential Decree No. 1529, a certificate of title shall not be subject to collateral
attack. It cannot be altered, modified or cancelled, except in a direct proceeding for that purpose in accordance
with law. The issue of the validity of the title of the respondents can only be assailed in an action expressly
instituted for that purpose.

Whether or not the petitioners have the right to claim ownership over the property is beyond the power of the
court a quo to determine in an action for unlawful detainer.”

ART 429

1.GERMAN MANAGEMENT & SERVICES VS CA

FACTS

The Spouses Jose were the owners of a parcel of land. They executed a special power of attorney authorizing German
Management & Services (German Management) to develop their property into a residential subdivision.

German Management found that part of the property was occupied by the private respondents and 20 other
persons. They advised the occupants to vacate the premises but the latter refused. Nevertheless, German
Management proceeded with the development of the property.

The private respondents filed an action for forcible entry against German Management. They alleged that German
Management forcibly removed and destroyed the barbed wire fence enclosing the private respondents farmholdings
without notice; bulldozed the rice, corn fruit bearing trees and other crops of private respondents by means of force,
violence and intimidation; trespassed, coerced and threatened to harass, remove and eject private respondents from
their respective farmholdings.

ISSUE/s

|
Whether German Management was justified in bulldozing and destroying the crops of private respondents on the
basis of the Doctrine of Self-Help.

RULING

No. German Management's drastic action of bulldozing and destroying the crops of private respondents on the basis
of the doctrine of self-help enunciated in Article 429 of the New Civil Code was unavailing because the doctrine of
self-help can only be exercised at the time of actual or threatened dispossession which is absent in the case at bar.

Art. 429. The owner or lawful possessor of a thing has the right to exclude any person from the enjoyment and
disposal thereof. For this purpose, he may use such force as may be reasonably necessary to repel or prevent an
actual or threatened unlawful physical invasion or usurpation of his property.

At the time German Management entered the property, the private respondents were already in possession thereof .
There was no evidence that the spouses Jose were ever in possession of the subject property. On the contrary,
private respondents' peaceable possession was manifested by the fact that they even planted rice, corn and fruit
bearing trees twelve to fifteen years prior to petitioner's act of destroying their crops.When possession has already
been lost, the owner must resort to judicial process for the recovery of property.

2. GRAND UNION SUPERMARKET, INC. and NELIA SANTOS FANDINO,vs.JOSE J. ESPINO JR., and THE HONORABLE COURT
OF APPEALS,

FACTS:
Plaintiff Jose J. Espino, Jr., a civil engineer and an executive of Procter and Gamble, was accused of intentionally
refusing to pay a “rat tail” file in one of the South Supermarket stores in Makati, as he forgot to pay such item
as his wife paid for all the grocery items they bought for the supermarket. The guards asked him to follow their
store procedures as to that situation, including signing of an incident report. As not to worsen the situation
caused by the procedures of the store, which was catching attention from other people from the store, petitioner
offered to pay defendant Nelia Santos-Fandino a P5.00 bill and said he was paying for the file whose cost was
P3.85. Fandino took the P5.00 bill from plaintiff stating that such is only the petitioner’s fine for having
committed the act. Plaintiff was shocked and objected vigorously that he was not a common criminal, and they
wanted to get back the P5.00. But Fandino told them that the money would be given as an incentive to the
guards who apprehend pilferers. The plaintiff then filed the present case to the trial to claim for damages of the
humiliation caused to him by the incident and to the fine collection, which, the plaintiff believes, the store owner
has no legal ground to collect such. After trial, the Court of First Instance of Pasig, Rizal dismissed the complaint,
interposing the appeal to the Court of Appeals, the latter reversed and set aside the appealed judgment, granting
damages, including exemplary damages for collecting a fine to the plaintiff. Not satisfied with the decision of
the respondent court, the petitioner instituted the present petition with the ground that there is no basis for her
to pay exemplary damages to the plaintiff.

Issue: Whether or not the petitioner legitimately exercised the right of defense of property within the
context of Article 429of the Civil Code and is thus exempted to pay the plaintiff with exemplary damages.

Held: Petitioners acted in good faith in trying to protect and recover their property, a right which the law
accords to them. Under Article 429, New Civil Code, the owner or lawful possessor of a thing has a right
to exclude any person from the enjoyment and disposal thereof and for this purpose, he may use such force as
may be reasonably necessary to repel or prevent an actual or threatened unlawful physical invasion or
usurpation of his property. And since a person who acts in the fulfillment of a duty or in the lawful exercise of a
right or office exempts him from civil or criminal liability, petitioner may not be punished by imposing fine
against the plaintiff or anyone who may cause damage to his property, or for that matter, to her store. We agree

|
that petitioners acted upon probable cause in stopping and investigating private respondent for taking the file
without paying for it, hence, the imposition of exemplary damages as a warning to others by way of a deterrent
is without legal basis. We, therefore, eliminate the grant of exemplary damages to the private respondent.

3. UNITED COCONUT PLANTERS BANK and LUIS MA.


ONGSIAPCO, vs. RUBEN E. BASCO
FACTS:
Respondent Ruben E. Basco had been employed with the petitioner United Coconut Planters Bank (UCPB)
for seventeen (17) years.[3] He was also a stockholder thereof and owned 804 common shares of stock at
the par value of P1.00.[4] He likewise maintained a checking account with the bank at its Las Pias Branch under
Account No. 117-001520-6.[5] Aside from his employment with the bank, the respondent also worked as an
underwriter at the United Coconut Planters Life Association (Coco Life), a subsidiary of UCPB since December,
1992.[6] The respondent also solicited insurance policies from UCPB employees.
On June 19, 1995, the respondent received a letter from the UCPB informing him of the termination of his
employment with the bank for grave abuse of discretion and authority, and breach of trust in the conduct of his
job as Bank Operations Manager of its Olongapo Branch. The respondent thereafter filed a complaint for illegal
dismissal, non-payment of salaries, and damages against the bank in the National Labor Relations Commission
(NLRC), docketed as NLRC Cases Nos. 00-09-05354-92 and 00-09-05354-93. However, the respondent still
frequented the UCPB main office in Makati City to solicit insurance policies from the employees thereat. He also
discussed the complaint he filed against the bank with the said employees.[7]
The respondent was also employed by All-Asia Life Insurance Company as an underwriter. At one time, the
lawyers of the UCPB had an informal conference with him at the head office of the bank, during which the
respondent was offered money so that the case could be amicably settled. The respondent revealed the incident
to some of the bank employees.[8]
On November 15, 1995, Luis Ma. Ongsiapco, UCPB First Vice-President, Human Resource Division, issued a
Memorandum to Jesus Belanio, the Vice-President of the Security Department, informing him that the
respondents employment had been terminated as of June 19, 1995, that the latter filed charges against the bank
and that the case was still on-going. Ongsiapco instructed Belanio not to allow the respondent access to all bank
premises.[9] Attached to the Memorandum was a passport-size picture of the respondent. The next day, the
security guards on duty were directed to strictly impose the security procedure in conformity with Ongsiapcos
Memorandum.[10]
On December 7, 1995, the respondent, through counsel, wrote Ongsiapco, requesting that such
Memorandum be reconsidered, and that he be allowed entry into the bank premises. [11]His counsel emphasized
that

In the meantime, we are more concerned with your denying Mr. Basco access to all bank premises. As you may
know, he is currently connected with Cocolife as insurance agent. Given his 17-year tenure with your bank, he
has established good relationships with many UCPB employees, who comprise the main source of his
solicitations. In the
course of his work as insurance agent, he needs free access to your bank premises, within reason, to add the
unnecessary. Your memorandum has effectively curtailed his livelihood and he is once again becoming a victim
of another illegal termination, so to speak. And Shakespeare said: You take his life when you do take the means
whereby he lives.

Mr. Bascos work as an insurance agent directly benefits UCPB, Cocolifes mother company. He performs his work
in your premises peacefully without causing any disruption of bank operations. To deny him access to your
premises for no reason except the pendency of the labor case, the outcome of which is still in doubt his liability,
if any, certainly has not been proven is a clear abuse of right in violation of our clients rights. Denying him access
to the bank, which is of a quasi-public nature, is an undue restriction on his freedom of movement and right to

|
make a livelihood, comprising gross violations of his basic human rights. (This is Human Rights Week,
ironically).

The respondent was undaunted. At 5:30 p.m. of December 21, 1995, he went to the office of JunneCacay, the
Assistant Manager of the Makati Branch. Cacay was then having a conference with Bong Braganza, an officer of
the UCPB Sucat Branch. Cacay entertained the respondent although the latter did have an appointment. Cacay
even informed him that he had a friend who wanted to procure an insurance policy.[14] Momentarily, a security
guard of the bank approached the respondent and told him that it was already past office hours. He was also
reminded not to stay longer than he should in the bank premises.[15] Cacay told the guard that the respondent
would be leaving shortly.[16] The respondent was embarrassed and told Cacay that he was already leaving.[17]
At 1:30 p.m. of January 31, 1996, the respondent went to the UCPB Makati Branch to receive a check from
Rene Jolo, a bank employee, and to deposit money with the bank for a friend.[18] He seated himself on a sofa
fronting the tellers booth[19] where other people were also seated.[20] Meanwhile, two security guards
approached the respondent. The guards showed him the Ongsiapcos Memorandum and told him to leave the
bank premises. The respondent pleaded that he be allowed to finish his transaction before leaving. One of the
security guards contacted the management and was told to allow the respondent to finish his transaction with
the bank.
Momentarily, Jose ReginoCasil, an employee of the bank who was in the 7th floor of the building, was asked
by Rene Jolo to bring a check to the respondent, who was waiting in the lobby in front of the tellers
booth.[21] Casil agreed and went down to the ground floor of the building, through the elevator. He was standing
in the working area near the Automated Teller Machine (ATM) Section[22] in the ground floor when he saw the
respondent standing near the sofa[23] near the two security guards.[24] He motioned the respondent to come and
get the check, but the security guard tapped the respondent on the shoulder and prevented the latter from
approaching Casil. The latter then walked towards the respondent and handed him the check from Jolo.
Before leaving, the respondent requested the security guard to log his presence in the logbook. The guard
did as requested and the respondents presence was recorded in the logbook.[25]

10. Plaintiff thereafter decided to contest his termination by filing an action for illegal dismissal against the
bank.

Despite the pendency of this litigation, plaintiff was reported visiting employees of the bank in their place of
work during work hours, and circulating false information concerning the status of his case against the bank,
including alleged offers by management of a monetary settlement for his illegal dismissal.

11. Defendants acted to protect the banks interest by preventing plaintiffs access to the banks offices, and at the
same time informing him of that decision.

Plaintiff purported to insist on seeing and talking to the banks employees despite this decision, claiming he
needed to do this in connection with his insurance solicitation activities, but the bank has not reconsidered.

12. The complaint states, and plaintiff has, no cause of action against defendants.[29]

The petitioners likewise interposed compulsory counterclaims for damages.

ISSUE: Whether or not the appellate court erred when it found that UCPB excessively exercised its right to self-
help to the detriment of Basco as a depositor, when on January 31, 1996, its security personnel stopped
respondent from proceeding to the area restricted to UCPBs employees.

|
RULING: The petitioners aver that the petitioner bank has the right to prohibit the respondent from access to all
bank premises under Article 429 of the New Civil Code, which provides that:

Art. 429. The owner or lawful possessor of a thing has the right to exclude any person from the enjoyment and
disposal thereof. For this purpose, he may use such force as may be reasonably necessary to repel or prevent an
actual or threatened unlawful physical invasion or usurpation of his property.

The petitioners contend that the provision which enunciates the principle of self-help applies when there is
a legitimate necessity to personally or through another, prevent not only an unlawful, actual, but also a
threatened unlawful aggression or usurpation of its properties and records, and its personnel and
customers/clients who are in its premises. The petitioners assert that petitioner Ongsiapco issued his
Memorandum dated November 15, 1995 because the respondent had been dismissed from his employment for
varied grave offenses; hence, his presence in the premises of the bank posed a threat to the integrity of its
records and to the persons of its personnel. Besides, the petitioners contend, the respondent, while in the bank
premises, conversed with bank employees about his complaint for illegal dismissal against the petitioner bank
then pending before the Labor Arbiter, including negotiations with the petitioner banks counsels for an
amicable settlement of the said case.
The respondent, for his part, avers that Article 429 of the New Civil Code does not give to the petitioner
bank the absolute right to exclude him, a stockholder and a depositor, from having access to the bank premises,
absent any clear and convincing evidence that his presence therein posed an imminent threat or peril to its
property and records, and the persons of its customers/clients.
We agree with the respondent bank that it has the right to exclude certain individuals from its premises or
to limit their access thereto as to time, to protect, not only its premises and records, but also the persons of its
personnel and its customers/clients while in the premises. After all, by its very nature, the business of the
petitioner bank is so impressed with public trust; banks are mandated to exercise a higher degree of diligence in
the handling of its affairs than that expected of an ordinary business enterprise.[41] Banks handle transactions
involving millions of pesos and properties worth considerable sums of money. The banking business will thrive
only as long as it maintains the trust and confidence of its customers/clients. Indeed, the very nature of their
work, the degree of responsibility, care and trustworthiness expected of officials and employees of the bank is
far greater than those of ordinary officers and employees in the other business firms. [42] Hence, no effort must
be spared by banks and their officers and employees to ensure and preserve the trust and confidence of the
general public and its customers/clients, as well as the integrity of its records and the safety and well being of
its customers/clients while in its premises. For the said purpose, banks may impose reasonable conditions or
limitations to access by non-employees to its premises and records, such as the exclusion of non-employees
from the working areas for employees, even absent any imminent or actual unlawful aggression on or an
invasion of its properties or usurpation thereof, provided that such limitations are not contrary to the law.[43]
It bears stressing that property rights must be considered, for many purposes, not as absolute, unrestricted
dominions but as an aggregation of qualified privileges, the limits of which are prescribed by the equality of
rights, and the correlation of rights and obligations necessary for the highest enjoyment of property by the
entire community of proprietors.[44] Indeed, in Rellosa vs. Pellosis,[45] we held that:

Petitioner might verily be the owner of the land, with the right to enjoy and to exclude any person from the
enjoyment and disposal thereof, but the exercise of these rights is not without limitations. The abuse of rights
rule established in Article 19 of the Civil Code requires every person to act with justice, to give everyone his due;
and to observe honesty and good faith. When right is exercised in a manner which discards these norms
resulting in damage to another, a legal wrong is committed for which the actor can be held accountable.

Rights of property, like all other social and conventional rights, are subject to such reasonable limitations in
their enjoyment and to such reasonable restraints established by law.[46]

|
ART 432

TAN VS. STANDARD OIL

FACTS: Standard Vacuum Oil Company ordered the delivery to the Rural Transit Company 1,925 gallons of
gasoline using a gasoline tank-truck trailer. The truck was driven by JulitoSto. Domingo, who was helped
Igmidio Rico. While the gasoline was being discharged to the underground tank, it caught fire, whereupon
JulitoSto. Domingo drove the truck across the Rizal Avenue Extension and upon reaching the middle of the street
he abondoned the truck with continued moving to the opposite side of the first street causing the buildings on
that side to be burned and detroyed. The house of Anita Tan was among those destroyed.

JulitoSto. Domingo and Imigidio Rico charged with arson through reckless imprudence after trial, both were
acquitted, the court holding that their negligence was not proven and the fire was due to an unfortunate accident.

Anita Tan then brought the action against the Standard Vacuum Oil Company and the Rural Transit Company;,
Defendants filed separate motions to dismiss alleging in substance that plaintiff's action is barred by prior
judgment. And the rule that extinction of the penal action does not carry with it extinction of the civil, unless the
extinction proceeds from the declaration in a final judgment that the fact from which the civil might arise did not
exist.

ISSUE: WON the TAN can still go after Vacuum Oil and Rural Transit

HELD: YES,

When the court acquitted the accused because the fire was due to an unfortunate accident it actually said that
the fire was due to a fortuitous event for which the accused are not to blame. It actually exonerated them from
civil liability. But the case takes a different aspect with respect to the other defendants. For one thing, the
principle of res judicata cannot apply to them for the simple reason that they were not included as co-accused in
the criminal case.

Take, for instance, of the Standard Vacuum Oil Company. this company is sued not precisely because of supposed
negligent acts of its two employees JulitoSto. Domingo and Igmidio Rico but because of acts of its own which
might have contributed to the fire that destroyed the house of the plaintiff. The precautions or measures which
this company has allegedly failed to take to prevent fire are not clearly stated, but they are matters of evidence
which need not now be determined. Suffice it to say that such allegation furnishes enough basis for a cause of
action against this company. There is no need for the plaintiff to make a reservation of her right to file a separate
civil action

Second. In cases where the persons for whose benefit the harm has been prevented shall be civilly liable in
proportion to the benefit which they have received.

Company can hardly be disputed, it appearing that the damage caused to the plaintiff was brought about mainly
because of the desire of driver JulitoSto. Domingo to avoid greater evil or harm, which would have been the case
had he not brought the tank-truck trailer to the middle of the street, for then the fire would have caused the

|
explosion of the gasoline deposit of the company which would have resulted in a conflagration of much greater
proportion and consequences to the houses nearby or surrounding it. It cannot be denied that this company is one
of those for whose benefit a greater harm has been prevented, and as such it comes within the purview of said
penal provision. The acquittal of the accused cannot, therefore, be deemed a bar to a civil action against this
company because its civil liability is completely divorced from the criminal liability of the accused. The rule
regarding reservation of the right to file a separate civil action does not apply to it.

Cornelio del Fierro, et. al, petitioners

vs

Rene Seguiran, respondent

FACTS:
The subject of this case are two parcels of agricultural land, Lot Nos. 1625 and 1626. The records of the Lands
Management Bureau, show that the claimants of Lot No. 1625 was LodelfoMarcial versus Miguel del Fierro,
while the claimants of Lot No. 1626 were LodelfoMarcial versus Francisco Santos and NarcisoMarcial.

 August 1964: the heirs of Miguel del Fierro, filed an ejectment case (forcible entry) against LodelfoMarcial and
NarcisoMarcial before the MTC. Judgment was rendered in favor of del Fierros which was affirmed by RTC.
 June 1964:LodelfoMarcial mortgaged to the Rural Bank of San Marcelino, Inc.
 December 1972: The bank extrajudicially foreclosed the real estate mortgage and was the highest bidder in the
sale of the property per the Certificate of Sale issued by the Provincial Sheriff.
 October 1981,Marcial executed in favor of respondent Rene Seguiran a Deed of Absolute Sale over a parcel of
swampland designated as Lot Nos. 1625 and 1626
 November 1981: Seguiran purchased Marcial's foreclosed property from the Rural Bank of San Marcelino Inc.
Respondent then filed an application for free patent over Lot Nos. 1625 and 1626, which was approved by the
Bureau of Lands.
 August 1983: petitioner petitioned the RTC to conduct a relocation survey of Lot Nos. 1625 and 1626, which
petition was approved by the court.
 February 16, 1985: the heirs of Miguel and Generosa del Fierro filed a Motion to Quash Order of Execution
claiming they are in actual physical possession of Lot Nos. 1625 and 1626, and that prior to the sale of the said
lots to respondent, the vendor, LodelfoMarcial no longer had any right over the property, since he lost in Civil
Case No. 706-1 for ejectment filed by the Del Fierros.
 May 1988: defendant claimed that when he bought the land in dispute on October, 28, 1981, Marcial was no
longer its owner, but the Rural Bank of San Marcelino since Marcial failed to redeem the land within the one-
year period of redemption. His only purpose for buying the land from the mortgagor, LodelfoMarcial in
November 1981 was for the peaceful turn-over of the property to him by Marcial. [held in abeyance]
 April 1998: the trial court rendered judgment in favor of Seguiran which was then affirmed by CA. Hence this
petition.

ISSUE: WON the action for reconveyance filed by the petitioners must be granted.

HELD: NO.

Article 434 of the Civil Code provides that to successfully maintain an action to recover the ownership of
a real property, the person who claims a better right to it must prove two (2) things: first, the identity of the land
claimed; and second, his title thereto.

Petitioners filed an action for reconveyance and cancellation of titles. Hence, it was incumbent on
petitioners to prove the requisites of reconveyance, one of which is to establish the identity of the parcels of land
petitioners are claiming. To reiterate, in an accionreinvindicatoria, the person who claims that he has a better
right to the property must first fix the identity of the land he is claiming by describing the location, area and
boundaries thereof. Petitioners' failure to present sufficient evidence on the identity of the properties sought to be

|
recovered and their title thereto resulted in the dismissal of their complaint. Further, the documents presented
by petitioners to prove their title over Lot Nos. 1625 and 1626 showed that the properties covered therein were
located in Barrio Liozon, Palauig, Zambales, while Lot Nos. 1625 and 1626 are located in Barrio Locloc, Palauig,
Zambales.

Jakosalem and Dulfo v. Barangan

FACTS:

 Respondent Col. Roberto S. Barangan entered into a Land Purchase Agreement with Citadel Realty
Corporation. Upon full payment of the purchase price, a Deed of Absolute Sale was executed in his
favor.
o The TCT over the land was likewise transferred to his name.
 Barangan was unable to physically occupy the subject land as he was then a member of the
Philippine Air Force. Thus, he was often assigned to different stations in the Philippines.
 Upon his retirement from the government service, he went to visit his property and discovered that
it was being occupied by petitioner Godofredo B. Dulfo and his family.
 Barangan sent a letter to Dulfo demanding that they vacate the property.
o Dulfo, through his son-in-law Atty. Rogelio J. Jakosalem, replied that the said property was
owned by them through a Deed of Assignment from the previous owner, Samson.
 Barangan filed a complaint for violation of the anti-squatting law before the Barangay. No amicable
settlement was reached; hence, the complaint was filed before the Prosecutor’s Office.
o Case dismissed as the issue of ownership must be resolved in a civil action.
 Barangan filed a complaint for recovery of possession before the RTC of Antipolo, and presented the
following as evidence:
o Testimony of Estardo (caretaker of the subdivision) to the effect that the Dulfos were
previously occupying an adjacent lot, but then started squatting on Barangan’s property;
o Testimony of an employee of the Municipal Assessor of Rizal, to prove that Barangan was
the registered owner of the lot; and
o Testimony of Engr. Jonco, to prove that the lot owned by Barangan and that occupied by
Dulfo were one and the same.
 RTC: Dismissed the case. Prescription and laches already set in, and Barangan was not able to
sufficiently prove his claim.
 CA: Reversed.

ISSUES: 1) WON respondent Barangay is entitled to recover the subject project 2) WON Barangan is entitled to
compensation; 3) WON laches and prescription shall apply

HELD:

1) AFFIRMATIVE. Article 434 of the Civil Code provides that in an action to recover, the property must be
identified, and the plaintiff must rely on the strength of his title and not on the weakness of the defendant’s
claim." In other words, in order to recover possession, a person must prove (1) the identity of the land claimed,
and (2) his title.

In this case, respondent Barangan was able to prove the identity of the property and his title. To prove his title
to the property, he presented in evidence the following documents: (1) Land Purchase Agreement; (2) Deed of
Absolute Sale; (3) and a Torrens title registered under his name, TCT No. N-10772. To prove the identity of the
property, he offered the testimonies of Engr. Jonco, who conducted the relocation survey, and Estardo, the
caretaker of the subdivision, who showed respondent Barangan the exact location of the subject property. He
likewise submitted as evidence the Verification Survey Plan of Lot 11, Block 5, (LRC) Psd-60846, which was
plotted based on the technical description appearing on respondent Barangan’s title.

|
Neither is there any discrepancy between the title number stated in the Land Purchase Agreement and the Deed
of Absolute Sale. As correctly found by the CA, TCT No. 171453, the title stated in the Deed of Absolute Sale, is a
transfer from TCT No. 165456, the title stated in the Land Purchase Agreement. Hence, both TCTs pertain to the
same property.

2) Since respondent Barangan was deprived of possession of the subject property, he is entitled to
reasonable compensation in the amount of P3,000.00 per month from November 17, 1994, the date of judicial
demand, up to the time petitioners vacate the subject property. The legal interest of which shall be at the rate of
6% per annum from November 17, 1994 and at the rate of 12% per annum from the time the judgment of this
Court becomes final and executory until the obligation is fully satisfied.

3) Laches and prescription do not apply. We agree with the CA that these do not apply in the instant case.
Jurisprudence consistently holds that "prescription and laches can not apply to registered land covered by the
Torrens system" because "under the Property Registration Decree, no title to registered land in derogation to that
of the registered owner shall be acquired by prescription or adverse possession."

ART 435

G.R. No. 158563 June 30, 2005


AIR TRANSPORTATION OFFICE (ATO) and MACTAN-CEBU INTERNATIONAL AIRPORT AUTHORITY (mciaa)
vs.
APOLONIO GOPUCO, JR

FACTS:

the NAC informed the owners of the various lots surrounding the Lahug Airport, including the herein respondent, that
the government was acquiring their lands for purposes of expansion. Some landowners were convinced to sell their
properties on the assurance that they would be able to repurchase the same when these would no longer be used by
the airport. Others, including Respondent Gopuco, owner of Lot no. 72 refused to do so.

Civil Aeronautics Administration (CAA) filed a complaint CFI of Cebu for the expropriation of Lot No. 72 and its
neighboring realties

CFI promulgated the decision declaring the expropriation of [the subject lots, including Lot No. 72] justified and in lawful
exercise of the right of eminent domain

Subsequently, when the Mactan International Airport commenced operations, the Lahug Airport was ordered closed by
then President Corazon C. Aquino in a Memorandum of 29 November 1989.6 Lot No. 72 was thus virtually abandoned

ApolonioGopuco, Jr. filed an amended complaint11 for recovery of ownership of Lot No. 72 against the Air
Transportation Office12 and the Province of Cebu with the Regional Trial Court (RTC) of Cebu, Branch X, docketed as
Civil Case No. CEB-11914. He maintained that by virtue of the closure of the Lahug Airport, the original purpose for
which the property was expropriated had ceased or otherwise been abandoned, and title to the property had therefore
reverted to him.

ISSUE:
When private land is expropriated for a particular public use, and that particular public use is abandoned, does its
former owner acquire a cause of action for recovery of the property?

RULING: NO

|
The answer to that question depends upon the character of the title acquired by the expropriator, whether it be the
State, a province, a municipality, or a corporation which has the right to acquire property under the power of eminent
domain. If, for example, land is expropriated for a particular purpose, with the condition that when that purpose is
ended or abandoned the property shall return to its former owner, then, of course, when the purpose is
terminated or abandoned the former owner reacquires the property so expropriated. If, for example, land is
expropriated for a public street and the expropriation is granted upon condition that the city can only use it for a public
street, then, of course, when the city abandons its use as a public street, it returns to the former owner, unless there is
some statutory provision to the contrary. . . If upon the contrary, however, the decree of expropriation gives to the entity
a fee simple title, then of course, the land becomes the absolute property of the expropriator, whether it be the State, a
province, or municipality, and in that case the non-user does not have the effect of defeating the title acquired by the
expropriation proceedings.

When land has been acquired for public use in fee simple, unconditionally, either by the exercise of eminent
domain or by purchase, the former owner retains no rights in the land, and the public use may be abandoned
or the land may be devoted to a different use, without any impairment of the estate or title acquired, or any
reversion to the former owner.

Gopuco argues that there is present, in cases of expropriation, an "implied contract" that the properties will be used
only for the public purpose for which they were acquired. No such contract exists.

Eminent domain is generally described as "the highest and most exact idea of property remaining in the government"
that may be acquired for some public purpose through a method in the nature of a forced purchase by the State.32 Also
often referred to as expropriation and, with less frequency, as condemnation, it is, like police power and taxation, an
inherent power of sovereignty and need not be clothed with any constitutional gear to exist; instead, provisions in our
Constitution on the subject are meant more to regulate, rather than to grant, the exercise of the power. It is a right to
take or reassert dominion over property within the state for public use or to meet a public exigency and is said to be an
essential part of governance even in its most primitive form and thus inseparable from sovereignty. 33 In fact, "all
separate interests of individuals in property are held of the government under this tacit agreement or implied
reservation. Notwithstanding the grant to individuals, the eminent domain, the highest and most exact idea of property,
remains in the government, or in the aggregate body of people in their sovereign capacity; and they have the right
to resume the possession of the property whenever the public interest so requires it."34

The ubiquitous character of eminent domain is manifest in the nature of the expropriation proceedings. Expropriation
proceedings are not adversarial in the conventional sense, for the condemning authority is not required to assert any
conflicting interest in the property. Thus, by filing the action, the condemnor in effect merely serves notice that it is
taking title and possession of the property, and the defendant asserts title or interest in the property, not to prove a
right to possession, but to prove a right to compensation for the taking. 35

The only direct constitutional qualification is thus that "private property shall not be taken for public use without just
compensation."36 This prescription is intended to provide a safeguard against possible abuse and so to protect as well
the individual against whose property the power is sought to be enforced.

In this case, the judgment on the propriety of the taking and the adequacy of the compensation received have long
become final. We have also already held that the terms of that judgment granted title in fee simple to the Republic of
the Philippines. Therefore, pursuant to our ruling in Fery, as recently cited in Reyes v. National Housing Authority,38no
rights to Lot No. 72, either express or implied, have been retained by the herein respondent.

|
REPUBLIC v. LIM

GR No. 161656

FACTS:

In 1938, the Republic instituted a special civil action for expropriation of a land in Lahug, Cebu City for the purpose of
establishing a military reservation for the Philippine Army. The said lots were registered in the name of Gervasia and
Eulalia Denzon. The Republic deposited P9,500 in the PNB then took possession of the lots. Thereafter, on May 1940,
the CFI rendered its Decision ordering the Republic to pay the Denzons the sum of P4,062.10 as just compensation.
The Denzons appealed to the CA but it was dismissed on March 11, 1948. An entry of judgment was made on April 5,
1948.

In 1950, one of the heirs of the Denzons, filed with the National Airports Corporation a claim for rentals for the two lots,
but it "denied knowledge of the matter." On September 6, 1961, Lt. Cabal rejected the claim but expressed willingness
to pay the appraised value of the lots within a reasonable time.

For failure of the Republic to pay for the lots, on September 20, 1961, the Denzons· successors-in-interest,Valdehueza
and Panerio, filed with the same CFI an action for recovery of possession with damages against the Republic and AFP
officers in possession of the property.

On November 1961, Titles of the said lots were issued in the names of Valdehueza and Panerio with the annotation
"subject to the priority of the National Airports Corporation to acquire said parcels of land, Lots 932 and939 upon
previous payment of a reasonable market value".

On July 1962, the CFI promulgated its Decision in favor of Valdehueza and Panerio, holding that they are the owners
and have retained their right as such over lots because of the Republic·s failure to pay the amount of
P4,062.10,adjudged in the expropriation proceedings. However, in view of the annotation on their land titles, they were
ordered to execute a deed of sale in favor of the Republic.

They appealed the CFI·s decision to the SC. The latter held that Valdehueza and Panerio are still the registered
owners of Lots 932 and 939, there having been no payment of just compensation by the Republic. SC still ruled that
they are not entitled to recover possession of the lots but may only demand the payment of their fair market value.

Meanwhile, in 1964, Valdehueza and Panerio mortgaged Lot 932 to Vicente Lim, herein respondent, as security for
their loans. For their failure to pay Lim despite demand, he had the mortgage foreclosed in 1976. The lot title was
issued in his name.

On 1992, respondent Lim filed a complaint for quieting of title with the RTC against the petitioners herein. On 2001, the
RTC rendered a decision in favor of Lim, declaring that he is the absolute and exclusive owner of the lot with all the
rights of an absolute owner including the right to possession. Petitioners elevated the case to the CA. In its Decision
dated September 18, 2003, it sustained the RTC Decision saying: ´... This is contrary to the rules of fair play because
the concept of just compensation embraces not only the correct determination of the amount to be paid to the owners
of the land,but also the payment for the land within a reasonable time from its taking. Without prompt payment,
compensation cannot be considered "just"...”

Petitioner, through the OSG, filed with the SC a petition for review alleging that they remain as the owner of Lot 932.

ISSUE:

Whether the Republic has retained ownership of Lot 932 despite its failure to pay respondent’s predecessors-in-
interest the just compensation therefor pursuant to the judgment of the CFI rendered as early as May 14, 1940.

HELD:

One of the basic principles enshrined in our Constitution is that no person shall be deprived of his private property
without due process of law; and in expropriation cases, an essential element of due process is that there must be just
compensation whenever private property is taken for public use. Accordingly, Section 9, Article III, of our Constitution
mandates: "Private property shall not be taken for public use without just compensation." The Republic disregarded the

|
foregoing provision when it failed and refused to pay respondent’s predecessors-in-interest the just compensation for
Lots 932 and 939.

The Court of Appeals is correct in saying that Republic’s delay is contrary to the rules of fair play. In jurisdictions similar
to ours, where an entry to the expropriated property precedes the payment of compensation, it has been held that if the
compensation is not paid in a reasonable time, the party may be treated as a trespasser ab initio.

As early as May 19, 1966, in Valdehueza, this Court mandated the Republic to pay respondent’s predecessors-in-
interest the sum of P16,248.40 as "reasonable market value of the two lots in question." Unfortunately, it did not
comply and allowed several decades to pass without obeying this Court’s mandate. It is tantamount to confiscation of
private property. While it is true that all private properties are subject to the need of government, and the government
may take them whenever the necessity or the exigency of the occasion demands, however from the taking of private
property by the government under the power of eminent domain, there arises an implied promise to compensate the
owner for his loss.

There is a recognized rule that title to the property expropriated shall pass from the owner to the expropriator only upon
full payment of the just compensation. So, how could the Republic acquire ownership over Lot 932 when it has not paid
its owner the just compensation, required by law, for more than 50 years? Clearly, without full payment of just
compensation, there can be no transfer of title from the landowner to the expropriator.

SC ruled in earlier cases that expropriation of lands consists of two stages. First is concerned with the determination of
the authority of the plaintiff to exercise the power of eminent domain and the propriety of its exercise. The second is
concerned with the determination by the court of "the just compensation for the property sought to be taken." It is only
upon the completion of these two stages that expropriation is said to have been completed In Republic v. Salem
Investment Corporation, we ruled that, "the process is not completed until payment of just compensation." Thus, here,
the failure of the Republic to pay respondent and his predecessors-in-interest for a period of 57 years rendered the
expropriation process incomplete.

Thus, SC ruled that the special circumstances prevailing in this case entitle respondent to recover possession of the
expropriated lot from the Republic.

While the prevailing doctrine is that "the non-payment of just compensation does not entitle the private landowner to
recover possession of the expropriated lots, however, in cases where the government failed to pay just compensation
within five (5) years from the finality of the judgment in the expropriation proceedings, the owners concerned shall have
the right to recover possession of their property. After all, it is the duty of the government, whenever it takes property
from private persons against their will, to facilitate the payment of just compensation. In Cosculluela v. Court of
Appeals, we defined just compensation as not only the correct determination of the amount to be paid to the property
owner but also the payment of the property within a reasonable time. Without prompt payment, compensation cannot
be considered "just.

NATIONAL POWER CORPORATION


vs.
HEIRS OF MACABANGKIT SANGKAY, namely: CEBU, BATOWA-AN, SAYANA, NASSER, MANTA, EDGAR,
PUTRI , MONGKOY*, and AMIR, all surnamed MACABANGKIT
FACTS:
Pursuant to Republic Act No. 6395, NPC undertook the Agus River Hydroelectric Power Plant Project in the
1970s to generate electricity for Mindanao.

The respondents, as the owners of land sued NPC in the RTC for the recovery of damages and of the property,
with the alternative prayer for the payment of just compensation

CONTENTION OF RESPONDENTS
They alleged that they had belatedly discovered that one of the underground tunnels of NPC that diverted the
water flow of the Agus River for the operation of the Hydroelectric Project in Agus V, Agus VI and Agus VII
traversed their land; that their discovery had occurred in 1995 after Atty. Saidali C. Gandamra, President of the
Federation of Arabic Madaris School, had rejected their offer to sell the land because of the danger the
underground tunnel might pose to the proposed Arabic Language Training Center and Muslims Skills

|
Development Center; that such rejection had been followed by the withdrawal by Global Asia Management and
Resource Corporation from developing the land into a housing project for the same reason; that Al-Amanah
Islamic Investment Bank of the Philippines had also refused to accept their land as collateral because of the
presence of the underground tunnel; that the underground tunnel had been constructed without their knowledge
and consent; that the presence of the tunnel deprived them of the agricultural, commercial, industrial and
residential value of their land; and that their land had also become an unsafe place for habitation because of the
loud sound of the water rushing through the tunnel and the constant shaking of the ground, forcing them and their
workers to relocate to safer grounds.

CONTENTION OF PETITIONER
NPC countered that the Heirs of Macabangkit had no right to compensation under section 3(f) of Republic Act
No. 6395, under which a mere legal easement on their land was established; that their cause of action, should
they be entitled to compensation, already prescribed due to the tunnel having been constructed in 1979; and that
by reason of the tunnel being an apparent and continuous easement, any action arising from such easement
prescribed in five years.

The RTC found that NPC had concealed the construction of the tunnel in 1979 from the Heirs of Macabangkit, and had
since continuously denied its existence ->CA affirmed

ISSUE:
WON NPC IS LIABLE FOR PAYMENT OF JUST COMPENSATION

RULING: YES
NPC’s construction of the tunnel constituted taking of the land, and entitled owners to just compensation
The Court held in National Power Corporation v. Ibrahim that NPC was "liable to pay not merely an easement fee but
rather the full compensation for land" traversed by the underground tunnels, viz:

In disregarding this procedure and failing to recognize respondents’ ownership of the sub-terrain portion, petitioner took
a risk and exposed itself to greater liability with the passage of time. It must be emphasized that the acquisition of the
easement is not without expense. The underground tunnels impose limitations on respondents’ use of the property for
an indefinite period and deprive them of its ordinary use. Based upon the foregoing, respondents are clearly entitled to
the payment of just compensation. Notwithstanding the fact that petitioner only occupies the sub-terrain portion, it is
liable to pay not merely an easement fee but rather the full compensation for land. This is so because in this case, the
nature of the easement practically deprives the owners of its normal beneficial use. Respondents, as the owner of the
property thus expropriated, are entitled to a just compensation which should be neither more nor less, whenever it is
possible to make the assessment, than the money equivalent of said property.

Here, like in National Power Corporation v. Ibrahim, NPC constructed a tunnel underneath the land of the Heirs of
Macabangkit without going through formal expropriation proceedings and without procuring their consent or at least
informing them beforehand of the construction. NPC’s construction adversely affected the owners’ rights and interests
because the subterranean intervention by NPC prevented them from introducing any developments on the surface,
and from disposing of the land or any portion of it, either by sale or mortgage.

Did such consequence constitute taking of the land as to entitle the owners to just compensation?

We agree with both the RTC and the CA that there was a full taking on the part of NPC, notwithstanding that the
owners were not completely and actually dispossessed. It is settled that the taking of private property for public use, to
be compensable, need not be an actual physical taking or appropriation. Indeed, the expropriator’s action may be
short of acquisition of title, physical possession, or occupancy but may still amount to a taking. Compensable taking
includes destruction, restriction, diminution, or interruption of the rights of ownership or of the common and necessary
use and enjoyment of the property in a lawful manner, lessening or destroying its value. It is neither necessary that the
owner be wholly deprived of the use of his property, nor material whether the property is removed from the possession
of the owner, or in any respect changes hands.

|
As a result, NPC should pay just compensation for the entire land. In that regard, the RTC pegged just compensation
at ₱500.00/square meter based on its finding on what the prevailing market value of the property was at the time of the
filing of the complaint, and the CA upheld the RTC

ART. 437

1. NAPOCOR VS. IBRAHIM

Facts:

Ibrahim owns a parcel of land located in Lanao del Norte.

In 1978, NAPOCOR took possession of the sub-terrain area of the land and constructed underground tunnels on
the said property.

The tunnels were apparently being used by NAPOCOR in siphoning the water of Lake Lanao and in the
operation of NAPOCOR’s Agus projects.

In 1991, Maruhom (one of the co-heirs of Ibrahim) requested Marawi City Water District for a permit to construct
or install a motorized deep well on the parcel of land but it was rejected on the grounds that the construction
would cause danger to lives and property by reason of the presence of the underground tunnels.

Maruhom demanded NAPOCOR to pay damages and to vacate the sub-terrain portion of the land.

Issue: WON Ibrahim is the rightful owner of the sub-terrain area of the land.

If yes, are they entitled to the payment of just compensation.

Held: YES. The sub-terrain portion of the property belongs to Ibrahim.

The Supreme Court cited Article 437 of the Civil Code which provides that: The owner of a parcel of land is the
owner of its surface and of everything under it, and he can construct thereon any works or make any plantations
and excavations which he may deem proper, without detriment to servitudes and subject to special laws and
ordinances. xxx

Hence, the ownership of land extends to the surface as well as to the subsoil under it. Therefore, Ibrahim owns
the property as well as the sub-terrain area of the land where the underground tunnels were constructed.

On the issue of just compensation, the Supreme Court also said that Ibrahim should be paid a just compensation.

Ibrahim could have dug upon their property and built motorized deep wells but was prevented from doing so by
the authorities because of the construction of the tunnels underneath the surface of the land.

Ibrahim still had a legal interest in the sub-terrain portion insofar as they could have excavated the same for the
construction of the deep wells. It has been shown that the underground tunnels have deprived the plaintiffs of
the lawful use of the land and considerably reduced its value.

It was held that: If the government takes property without expropriation and devotes the property to public use,
after many years, the property owner may demand payment of just compensation in the event restoration of
possession is neither convenient nor feasible. This is in accordance with the principle that persons shall not be
deprived of their property except by competent authority and for public use and always upon payment of just
compensation.

RIGHT OF ACCESSION

|
ART. 445

3. FRANCISCO MUNOZ JR. VS. ERLINDA RAMIREZ

FACTS:

Respondent-spouses mortgaged a residential lot (which the wife inherited) to the GSIS to secure a housing loan
(200k). Thereafter, they used the money loaned to construct a residential house on said lot.

It is alleged that MUNOZ granted the spouses a 600k loan, which the latter used to pay the debt to GSIS. The
balance of the loan (400k) will be delivered by MUNOS upon surrender of the title over the property and an
affidavit of waiver of rights (over the property) to be executed by the husband. While the spouses were able to
turn over the title, no affidavit was signed by the husband. Consequently, MUNOZ refused to give the 400k
balance of the loan and since the spouses could no longer return the 200k (which was already paid to GSIS),
MUNOZ kept the title over the property and subsequently, caused the issuance of a new one in his own name.

The spouses then filed a case for the annulment of the purported sale of the property in favor of MUNOZ. The
RTC ruled that the property was the wife’s exclusive paraphernal property (since she inherited it from her father)
and as such, the sale is valid even without the husband’s consent.

The CA reversed and ruled that while the property was originally exclusive paraphernal property of the wife, it
became conjugal property when it was used as a collateral for a housing loan that was paid through conjugal
funds. Hence, the sale is void.

ISSUE (1): Is the property paraphernal or conjugal?

RULING: PARAPHERNAL. As a general rule, all property acquired during the marriage is presumed to be
conjugal unless the contrary is proved. In this case, clear evidence that the wife inherited the lot from her father
has sufficiently rebutted this presumption of conjugal ownership. Consequently, the residential lot is the wife’s
exclusive paraphernal property (pursuant to Article 92 and 109 of FC).

It was error for the CA to apply Article 158 of the CC and the ruling on Calimlim-Canullas. True, respondents
were married during the effectivity of the CC and thus its provisions should govern their property relations. With
the enactment of the FC however, the provisions of the latter on conjugal partnership of gains superseded those
of the CC. Thus, it is the FC that governs the present case and not the CC. And under Article 120 of the FC
(which supersedes Article 158 of the CC), when the cost of the improvement and any resulting increase in the
value are more than the value of the property at the time of the improvement, the entire property shall belong to
the conjugal partnership, subject to reimbursement; otherwise, the property shall be retained in ownership by the
owner-spouse, likewise subject to reimbursement for the cost of improvement.

In this case, the husband only paid a small portion of the GSIS loan (60k). Thus, it is fairly reasonable to assume
that the value of the residential lot is considerably more than the contribution paid by the husband. Thus, the
property remained the exclusive paraphernal property of the wife at the time she contracted with MUNOZ; the
written consent of the husband was not necessary.

ISSUE (2): Was the transaction a sale or equitable mortgage?

RULING: EQUITABLE MORTGAGE. Under Article 1602 of the CC, a contract is presumed an equitable
mortgage when: (a) price of sale with right to repurchase is unusually inadequate; (b) vendor remains in
possession as lessee or otherwise; (c) upon or after the expiration of the right to repurchase, another instrument
extending the period of redemption is executed; (d) purchase retains for himself a part of the purchase price; (e)
vendor binds himself to pay the taxes on the thing sold; and, (f) in any other case it may be fairly inferred that
the real intention of the parties is for the transaction to secure the payment of a debt.

In this case, considering that (a) the spouses remained in possession of the property (albeit as lessees thereof); (b)
MUNOZ retained a portion of the ‘purchase price’ (200k); (c) it was the spouses who paid real property taxes on
the property; and, (d) it was the wife who secure the payment of the principal debt with the subject property —
the parties clearly intended an equitable mortgage and not a contract of sale.

|
ART 447

MARTIN VS MARTIN
FACTS
Jose Balagui and DoroteaBalagui, brother and sister, sold the two parcels of land subject of the action, to
Feliciano Martin and Florentino Martin for P1,200. Jose Balagui brought an actionagainst said Feliciano and
Florentino Martin for damages arising from failure of the Martins to comply with some conditions agreed upon
in the sale. The said action was terminated by a compromise agreement between Feliciano Martin, Florentino
Martin and Isidro Martin.
CA found that Feliciano Martin did in fact the sign agreement. The court also found that the intention of the
parties was to transform the original sale made in favor of Feliciano and Florentino Martin into an equitable
mortgage, as contended by the spouses and their transferees, the defendant Prudencio Martin and intervenor
Ignacio de la Cruz. The court also found that Jose Balagui sold the parcels of land in question to Ignacio de la
Cruz for the sum of P2,500, with the understanding that the purchaser would redeem the lands from Feliciano
Martin and Florentino Martin by paying to them the sum of P1,200.
CA reversed the decision of the CFI which had declared the compromise null and void for having been made
before a court which had no competent jurisdiction over the action.
Petitioner contending that the Court of Appeals erred in declaring that the compromise had the effect of
converting the previous contract of sale into one of loan secured by a mortgage; and on failing to make a finding
on the rights and obligations of the petitioner, with respect to the houses builts on the lands in good faith by the
petitioner Feliciano Martin and hi son-in-law and his daughter. The court of Appeals is also alleged to have made
an error in declaring that the compromise was valid even if the court before which it was made had no
jurisdiction over the case brought and in which it was entered into.
ISSUES:
1. W/N the compromise agreement have the effect of converting the sale in into loan secured by mortgaged
2. W/N the CA failed to make a finding on the rights and obligations of the petitioner, with respect to the houses
builts on the lands in good faith by the petitioner Feliciano Martin and hi son-in-law and his daughter.
HELD
1. NO. petitioner Feliciano Martin had actually signed the compromise agreement, this being a finding of fact,
which is final and binding upon us. A person cannot repudiate the effects of his voluntary acts simply because it
does not fit him, or simply because the judge before whom he executed the act did not have jurisdiction of the
case. In a regime of law and order, repudiation of an agreement validly entered into can not be made without
any ground or reason in law or in fact for such repudiation. The conclusion of the trial court in respect to the
validity of the compromise agreement and its binding effect upon Feliciano Martin cannot be questioned.
2. Yes. The decision of the Court of Appeals is silent on the rights and obligations of the parties with respect to
the said houses. We find merit in the contention that the Court of Appeals erred in failing to make a specific
pronouncement on the rights and obligations of the parties with respect to the said houses.
The Court of Appeals found that the houses were built after October 31, 1930, after Feliciano Martin had
returned the amount of P600 that Florentino Martin had contributed to the purchase money. At the time of the
construction, therefore, the petitioner had already become the rightful possessor of the land having, besides,
declared them for tax purposes. No claim is made by any of the parties-respondents that the construction of the
houses had been made in bad faith. The compromise agreement did not specify within what period of time
Feliciano Martin was to enjoy the possession and use of the lands in question. Neither has there any evidence
submitted to show that the building of the houses was prohibited by the original owners of the land or by the
subsequent purchaser. A portion of the land was residential, so its use could only be enjoyed by the building of a

|
house thereon. So we must find as a fact that the building of the houses was made in good faith and in the
exercise of the rights granted to Feliciano Martin by the compromise agreement.
The law applicable to petitioner is Article 361 of the Spanish Civil Code, which provides as follows:
Art. 361. The owner of land on which anything has been built sown, or planted, in good faith, shall be entitled to
appropriate the thing so built, sown, or planted, upon paying the indemnification mentioned in Articles 453 and
454, or to compel the person who has built or planted to pay him the value of the land, and the person who
sowed thereon to pay the proper rent therefor.
Ignacio de la Cruz is declared to be the owner of the lands subjects of the action and entitled to the possession
thereof upon payment by him of the sum of P600 to petitioner Feliciano Martin, but the decision is modified by
further ordering that the case be remanded to the court below for determination of the price or the value of the
two houses built on the lands in question, and thereafter for the intervenor-appellee to exercise the option
specified in Article 361 of the Spanish Civil Code.
ART. 448
1. Benedicto vs. Villaflores
FACTS:

 Maria Villaflores (Maria) was the owner of Lot 2-A, with an area of 277 square meters, in Bulacan. In
1980, Maria sold a portion of Lot 2-A to her nephew, Antonio Villaflores.

 Antonio then took possession of the portion sold to him and constructed a house thereon.

 Twelve (12) years later, Maria executed in favor of Antonio a Kasulatan ng BilihangTuluyan covering the
entire Lot 2-A. However, Antonio did not register the sale or pay the real property taxes for the subject
land.

 In 1994, Maria sold the same Lot 2-A to Filomena. Filomena registered the sale with the Registry of
Deeds. Consequently, Transfer of Certificate of Title was issued in the name of Filomena. Since then
Filomena paid the real property taxes for the subject parcel of land.

 In September 2000, Filomena filed a case for AccionPubliciana with Cancellation of Notice of Adverse
Claimagainst Antonio.

 She alleged at the time of the sale, she was not aware that Antonio had any claim or interest over the
subject property.

 Antonio assured her that there was no impediment to her acquisition of the land, and promised to vacate
the property five (5) years after the sale. In August 1999, Antonio requested an extension of one (1)
year, and offered to pay a monthly rental which she granted. However, in 2000, Antonio refused to
vacate the property and, instead, claimed absolute ownership of Lot 2-A.

 Antonio asserted absolute ownership over Lot 2-A. He alleged that he purchased the subject property.
He averred that Filomena was aware of the sale; hence, the subsequent sale in favor of Filomena was
rescissible, fraudulent, fictitious, or simulated.

 RTC rendered a decision sustaining Filomena’s ownership. According to the RTC, Filomena was the one
who registered the sale in good faith; as such, she has better right than Antonio. It rejected Antonio’s
allegation of bad faith on the part of Filomena because no sufficient evidence was adduced to prove it. It
declared Antonio a builder in good faith.

|
 CA affirmed the decision of the RTC and remanded the case to the RTC for further proceedings to
determine the respective rights of the parties under Articles 448 and 546 of the Civil Code.

ISSUE: WON Antonio is a builder in good faith


HELD: YES.

It is not disputed that the construction of Antonio’s house was undertaken long before the sale in favor
of Filomena; that when Filomena bought the property from Maria, Antonio’s house which he used as residence
had already been erected on the property.

Thus, we sustain the finding that Antonio is a builder in good faith.

Under Article 448, a landowner is given the option to either appropriate the improvement as his own upon
payment of the proper amount of indemnity, or sell the land to the possessor in good faith. Relatedly, Article
546 provides that a builder in good faith is entitled to full reimbursement for all the necessary and useful
expenses incurred; it also gives him right of retention until full reimbursement is made.

The builder in good faith can compel the landowner to make a choice between appropriating the building by
paying the proper indemnity or obliging the builder to pay the price of the land. The choice belongs to the
owner of the land, a rule that accords with the principle of accession, i.e., that the accessory follows the principal
and not the other way around.

PEDRO ANGELES VS. ESTELITA PASCUAL

FACTS:

Neighbors Pascual and Angeles were registered owners of adjacent parcels of land located in Cabanatuan City.
Pascual owned Lot 4 consolidation-subdivision plan and Angeles owned Lot 5 of the same consolidation-
subdivision plan. Each of them built a house on his respective lot, believing all the while that his respective lot
was properly delineated. It was not until Metropolitan Bank and Trust Company (Metrobank), as the highest
bidder in the foreclosure sale of the adjacent Lot 3, Block 2 (Lot 3), caused the relocation survey of Lot 3 that the
geodetic engineer discovered that Pascual’s house had encroached on Lot 3. As a consequence, Metrobank
successfully ejected Pascual.

In turn, Pascual caused the relocation survey of his own Lot 4 and discovered that Angeles’ house also encroached
on his lot. Of the 318 square meters comprising Lot 4, Angeles occupied 252 square meters, leaving Pascual with
only about 66 square meters. Pascual demanded rentals for the use of the encroached area of Lot 4 from Angeles,
or the removal of Angeles’ house. Angeles refused the demand. Accordingly, Pascual sued Angeles for recovery of
possession and damages in the RTC.

CA: Plaintiffs-appellees are ordered to exercise within thirty (30) days from the finality of this decision their
option to either buy the portion of defendant-appellant’s house on their Lot. No. 4, or to sell to defendant-
appellant the portion of their land on which his house stands. If plaintiffs-appellees elect to sell the land or buy
the improvement, the purchase price must be at the prevailing market price at the time of payment..

Issues: WON the options laid down by the CA, i.e., for Pascual either to buy the portion of Angeles’ house or to
sell to Angeles the portion of his land occupied by Angeles were contrary to its finding of good faith.

|
Ruling: YES

Angeles was a builder in good faith

Article 448. The owner of the land on which anything has been built, sown or planted in good faith, shall have the
right to appropriate as his own the works, sowing or planting, after payment of the indemnity provided for in
articles 546 and 548, or to oblige the one who built or planted to pay the price of the land, and the one who sowed,
the proper rent. However, the builder or planter cannot be obliged to buy the land if its value is considerably
more than that of the building or trees. In such case, he shall pay reasonable rent, if the owner of the land does
not choose to appropriate the building or trees after proper indemnity.The parties shall agree upon the terms of
the lease and in case of disagreement, the court shall fix the terms thereof.

The provision contemplates a person building, or sowing, or planting in good faith on land owned by another. The
law presupposes that the land and the building or plants are owned by different persons, like here. The RTC and
CA found and declared Angeles to be a builder in good faith. We cannot veer away from their unanimous
conclusion, which can easily be drawn from the fact that Angeles insists until now that he built his house entirely
on his own lot. Good faith consists in the belief of the builder that the land he is building on is his and in his
ignorance of a defect or flaw in his title.15

With the unassailable finding that Angeles’ house straddled the lot of Pascual, and that Angeles had built his
house in good faith, Article 448 of the Civil Code, which spells out the rights and obligations of the owner of the
land as well as of the builder, is unquestionably applicable. Consequently, the land being the principal and the
building the accessory, preference is given to Pascual as the owner of the land to make the choice as between
appropriating the building or obliging Angeles as the builder to pay the value of the land. Contrary to the
insistence of Angeles, therefore, no inconsistency exists between the finding of good faith in his favor and the
grant of the reliefs set forth in Article 448.

[G.R. No. 149295. September 23, 2003]

PHILIPPINE NATIONAL BANK, petitioner, vs. GENEROSO DE JESUS

FACTS:

Respondent filed a complaint against petitioner before the Regional Trial Court of Occidental Mindoro for
recovery of ownership and possession, with damages, over the questioned property. On 26 March 1993, he had
caused a verification survey of the property and discovered that the northern portion of the lot was being
encroached upon by a building of petitioner to the extent of 124 square meters. Despite two letters of demand
sent by respondent, petitioner failed and refused to vacate the area.

Petitioner asserted that when it acquired the lot and the building sometime in 1981 from then Mayor Bienvenido
Ignacio, the encroachment already was in existence and to remedy the situation, Mayor Ignacio offered to sell the
area in question (which then also belonged to Ignacio) to petitioner at P100.00 per square meter which offer the
latter claimed to have accepted. The sale, however, did not materialize when, without the knowledge and consent
of petitioner, Mayor Ignacio later mortgaged the lot to the Development Bank of the Philippines.

The trial court decided the case in favor of respondent declaring him to be the rightful owner.

The Court of Appeals sustained the trial court.

ISSUE: W/N PETITIONER IS A BUILDER IN GOOD FAITH

HELD:

|
In reference to Article 448, et seq., of the Civil Code, a builder in good faith is one who, not being the owner of the
land, builds on that land believing himself to be its owner and unaware of any defect in his title or mode of
acquisition.

The essence of good faith lies in an honest belief in the validity of ones right, ignorance of a superior claim, and
absence of intention to overreach another. Applied to possession, one is considered in good faith if he is not aware
that there exists in his title or mode of acquisition any flaw which invalidates it. Evidently, petitioner was quite
aware, and indeed advised, prior to its acquisition of the land and building from Ignacio that a part of the
building sold to it stood on the land not covered by the land conveyed to it.

Equally significant is the fact that the building, constructed on the land by Ignacio, has in actuality been part of
the property transferred to petitioner. Article 448, of the Civil Code refers to a piece of land whose ownership is
claimed by two or more parties, one of whom has built some works (or sown or planted something) and not to a
case where the owner of the land is the builder, sower, or planter who then later loses ownership of the land by
sale or otherwise for, elsewise stated, where the true owner himself is the builder of works on his own land, the
issue of good faith or bad faith is entirely irrelevant.

In fine, petitioner is not in a valid position to invoke the provisions of Article 448 of the Civil Code.

NATIONAL HOUSING AUTHORITY vs. GRACE BAPTIST CHURCH and COURT OF APPEALS

FACTS: On June 13, 1986, Respondent Grace Baptist Church wrote a letter to NHA manifesting intent to
purchase Lot 4 and 17 of the General Mariano Alvarez Resettlement Project in Cavite. The latter granted request
hence respondent entered into possession of the lots and introduced improvements thereon. On February 22,
1991, NHA passed are solution approving the sale of the subject lots to respondent Church for 700 per square
meter, a total of P430,500. Respondents were duly informed.

On April 8, 1991, respondent church tendered a check amounting to P55,350 contending that this was the agreed
price. NHA avers stating that the price now (1991) is different from before (1986). The trial court rendered a
decision in favour of NHA stating that there was no contract of sale, ordering to return the said lots to NHA and
to pay NHA rent of 200 pesos from the time it took possession of the lot. Respondent Church appealed to the CA
which affirms the decision of RTC regarding “no contract of sale” but modifying it by ordering NHA to execute
the sale of the said lots to Church for 700 per square, with 6% interest per annum from March1991. Petitioner
NHA filed a motion for reconsideration which was denied. Hence this petition for review on certiorari.

ISSUE: WON NHA can be compelled to sell the lots under market value?

HELD: No, because the contract has not been perfected. The Church despite knowledge that its intended contract
of sale with the NHA had not been perfected proceeded to introduce improvements on the land. On the other
hand, NHA knowingly granted the Church temporary use of the subject properties and did not prevent the
Church from making improvements thereon.

Thus the Church and NHA, who both acted in bad faith, shall be treated as if they were both in good faith. In this
connection Art 448 provides: “The owner of the land in which anything has been built, sown or planted in good
faith, shall have the right to appropriate as his own the works, sowing or planting, after payment of the
indemnity provided for in articles 546 and 548, or to oblige the one who built or planted to pay the price of the
land, and the one who sowed, the proper rent. However, the builder or planter cannot be obliged to buy the land
and if its value is considerably more than that of the building or trees. In such case, he shall pay reasonable rent,
if the owner of the land does not choose to appropriate the building or trees after proper indemnity. The parties
shall agree, on case of disagreement, court shall fix.

SPOUSES JUAN NUGUID AND ERLINDA T. NUGUID


vs.
HON. COURT OF APPEALS AND PEDRO P. PECSON

|
FACTS:
Pedro P. Pecson owned a commercial lot on which he built a four-door two-storey apartment building.
For failure to pay realty taxes, the lot was sold at public auction to the spouses Juan and ErlindaNuguid.

Nuguids became the uncontested owners of the 256-square meter commercial lot.

As a result, the Nuguid spouses moved for delivery of possession of the lot and the apartment building.

Trial court, relying upon Article 546 of the Civil Code, ruled that the Spouses Nuguid were to reimburse Pecson for his
construction cost of ₱53,000, following which, the spouses Nuguid were entitled to immediate issuance of a writ of
possession over the lot and improvements. In the same order the RTC also directed Pecson to pay the same amount
of monthly rentals to the Nuguids as paid by the tenants occupying the apartment units or ₱21,000 per month from
June 23, 1993, and allowed the offset of the amount of ₱53,000 due from the Nuguids against the amount of rents
collected by Pecson from June 23, 1993 to September 23, 1993 from the tenants of the apartment
spouses Nuguid have not yet indemnified petitioner Pecson with the cost of the improvements.

The court ruled that the petitioner shall be restored to the possession of the apartment building until payment of the
required indemnity.

RULING:
Under Article 448, the landowner is given the option, either to appropriate the improvement as his own upon payment
of the proper amount of indemnity or to sell the land to the possessor in good faith. Relatedly, Article 546 provides that
a builder in good faith is entitled to full reimbursement for all the necessary and useful expenses incurred; it also gives
him right of retention until full reimbursement is made.

While the law aims to concentrate in one person the ownership of the land and the improvements thereon in view of
the impracticability of creating a state of forced co-ownership,23 it guards against unjust enrichment insofar as the
good-faith builder’s improvements are concerned. The right of retention is considered as one of the measures devised
by the law for the protection of builders in good faith. Its object is to guarantee full and prompt reimbursement as it
permits the actual possessor to remain in possession while he has not been reimbursed (by the person who defeated
him in the case for possession of the property) for those necessary expenses and useful improvements made by him
on the thing possessed.24 Accordingly, a builder in good faith cannot be compelled to pay rentals during the period of
retention25 nor be disturbed in his possession by ordering him to vacate. In addition, as in this case, the owner of the
land is prohibited from offsetting or compensating the necessary and useful expenses with the fruits received by the
builder-possessor in good faith. Otherwise, the security provided by law would be impaired. This is so because the
right to the expenses and the right to the fruits both pertain to the possessor, making compensation juridically
impossible; and one cannot be used to reduce the other. 26

As we earlier held, since petitioners opted to appropriate the improvement for themselves as early as June 1993, when
they applied for a writ of execution despite knowledge that the auction sale did not include the apartment building, they
could not benefit from the lot’s improvement, until they reimbursed the improver in full, based on the current market
value of the property.

Despite the Court’s recognition of Pecson’s right of ownership over the apartment building, the petitioners still insisted
on dispossessing Pecson by filing for a Writ of Possession to cover both the lot and the building. Clearly, this resulted
in a violation of respondent’s right of retention. Worse, petitioners took advantage of the situation to benefit from the
highly valued, income-yielding, four-unit apartment building by collecting rentals thereon, before they paid for the cost
of the apartment building. It was only four years later that they finally paid its full value to the respondent.

Petitioners’ interpretation of our holding in G.R. No. 115814 has neither factual nor legal basis. The decision of May 26,
1995, should be construed in connection with the legal principles which form the basis of the decision, guided by the
precept that judgments are to have a reasonable intendment to do justice and avoid wrong.

|
The text of the decision in G.R. No. 115814 expressly exempted Pecson from liability to pay rentals, for we found that
the Court of Appeals erred not only in upholding the trial court’s determination of the indemnity, but also in ordering him
to account for the rentals of the apartment building from June 23, 1993 to September 23, 1993, the period from entry of
judgment until Pecson’s dispossession. As pointed out by Pecson, the dispositive portion of our decision in G.R. No.
115814 need not specifically include the income derived from the improvement in order to entitle him, as a builder in
good faith, to such income. The right of retention, which entitles the builder in good faith to the possession as well as
the income derived therefrom, is already provided for under Article 546 of the Civil Code.

Given the circumstances of the instant case where the builder in good faith has been clearly denied his right of
retention for almost half a decade, we find that the increased award of rentals by the RTC was reasonable and
equitable. The petitioners had reaped all the benefits from the improvement introduced by the respondent during said
period, without paying any amount to the latter as reimbursement for his construction costs and expenses. They should
account and pay for such benefits.

Fernando Carrascoso v. Court of Appeals


G.R. No. 123672

FACTS
 El Dorado Plantation, Inc. (El Dorado) was the registered owner of a parcel of land (the property) with an area of
approximately 1,825 hectares covered by Transfer Certificate of Title (TCT) No. T-93 situated in Sablayan,
Occidental Mindoro.
 In March 1972, El Dorado Plantation, Inc. (El Dorado), through its board member Lauro Leviste, executed a Deed
of Sale with Fernando Carrascoso, Jr.
 The subject of the sale was a 1,825 hectare of land. It was agreed that Carrascoso was to pay P1.8M.; that P290K
would be paid by Carrascoso to PNB to settle the mortgage upon the said land. P210k would be paid directly to
Leviste. The balance of P1.3M plus 10% interest would be paid over the next 3 years at P519k every 25th of March.
Leviste also assured that there were no tenants hence the land does not fall under the Land Reform Code. Leviste
allowed Carrascoso to mortgage the land which the latter did.
 Carrascoso obtained a total of P1.07M as mortgage and he used the same to pay the down payment agreed upon
in the contract. Carrascoso defaulted from his obligation which was supposed to be settled on March 25, 1975.
Leviste then sent him letters to make good his end of the contract otherwise he will be litigated.
 In 1977, Carrascoso executed a Buy and Sell Contract with PLDT. The subject of the sale was the same land sold
to Carrascoso by Leviste but it was only the 1000 sq. m. portion thereof. The land is to be sold at P3M. Part of the
terms and conditions agreed upon was that Carrascoso is to remove all tenants from the land within one year. He
was also being given a 6-month extension in case he’ll need one. Thereafter, PLDT will notify Carrascoso if
whether or not PLDt will finalize the sale. Eventually, PLDT gained possession of the land.
 Meanwhile, El Dorado filed a civil case against Carrascoso. PLDT intervened averring that it was a buyer in good
faith. The Regional Trial Court (RTC) ruled in favor of Carrascoso. The Court of Appeals (CA) reversed the RTC
ruling.
ISSUE
What is the nature of each contract?
HELD
 The contract executed between El Dorado and Carrascoso was a contract of sale. It was perfected by their
meeting of the minds and was consummated by the delivery of the property to Carrascoso. However, El Dorado
has the right to rescind the contract by reason of Carrascoso’s failure to perform his obligation.
 A contract of sale is a reciprocal obligation. The seller obligates itself to transfer the ownership of and deliver a
determinate thing, and the buyer obligates itself to pay therefor a price certain in money or its equivalent. The non-
payment of the price by the buyer is a resolutory condition which extinguishes the transaction that for a time

|
existed, and discharges the obligations created thereunder. Such failure to pay the price in the manner prescribed
by the contract of sale entitles the unpaid seller to sue for collection or to rescind the contract.
 On the other hand, the contract between Carrascoso and PLDT is a contract to sell. This is evidenced by the terms
and conditions that they have agreed upon that after fulfillment of Carrascoso’s obligation PLDT has “to notify
Carrascoso of its decision whether or not to finalize the sale.”
Side Issue: Carrascoso also averred that there was a breach on El Dorado’s part when it comes to warranty.
Carrascoso claimed that there were tenants on the land and he spent about P2.9M relocating them. The SC ruled
that Carrascoso merely had a bare claim without additional proof to support it.
 Requisites of Express warranty in a Contract of Sale
(1) the express warranty must be an affirmation of fact or any promise by the seller relating to the subject matter of the
sale;
(2) the natural tendency of such affirmation or promise is to induce the buyer to purchase the thing; and
(3) the buyer purchases the thing relying on such affirmation or promise thereon.

SPS. ENRIQUETA RASDAS, and TOMAS RASDAS, SPS. ESPERANZA A. VILLA, and ERNESTO VILLA, and
LOLITA GALLE vs. JAIME ESTENOR
Facts: Respondents filed a complaint for recovery of ownership and possession with damages against petitioners for a
parcel of land with an area of 703 square meters in Isabela. The petitioners had built a house on the disputed lot.

The Court of Appeals ruled in favor of the respondents. The petitioners were late in appealing the decision. The CA
decision had already become final and executory and a writ of execution and demolition had already
been issued against them.

The petitioners conceded that they did not own the lot but alleged that they were entitled to reimbursement for the
value of the house they constructed. Furthermore, they alleged that even if they werebuilders in bad faith, the
respondent was an owner in bad faith and that the case should be treated as if both parties were in good faith.

The respondent, on the other hand, argued res judicata in that the issue had already been ruled upon by the CA finding
them buildersin bad faith and thereby builders in bad faith.

Issue: Is the petition barred by res judicata such that petitioners are no longer entitled to reimbursement for their
house on respondent’s lot?

Held: Although there was some error in the process followed by the lower courts, the petition is barred by res judicata.
The CA found that the occupancy by the petitioners was merely tolerated by the respondent, who was verbally begged
by the father of the petitioners to allow them to remain on the disputed lot as the petitioners were still studying. As for
the respondent being an owner in bad faith, the matter was raised only in its petition to the Supreme Court and could
not be entertained as a matter of procedure. Furthermore, nothing in the rulings of the lower courts even supports such
a proposition.

You might also like